Representación de dimensión infinita del álgebra de Lorentz

En QFT, necesitamos usar representaciones de dimensión infinita del álgebra de Lorentz, porque todas las representaciones de dimensión finita no triviales no son unitarias, y necesitamos representaciones unitarias para que ψ | ψ es invariante bajo las transformaciones de Lorentz (Schwartz, Quantum Field Theory, pag 110),

ψ | ψ ψ | Λ Λ | ψ = ψ | ψ
si Λ Λ = I .

Las representaciones de dimensión infinita del álgebra de Lorentz se logran mediante operadores diferenciales (Maggiore, A Modern Introduction to Quantum Field Theroy, pág. 30, y Zee, Group Theory, pág. 434), que actúan sobre el espacio de Hilbert de funciones integrables al cuadrado. (que es de dimensión infinita), en lugar de matrices que actúan sobre el R 4 espacio vectorial

Así que entiendo que el objetivo de las representaciones de dimensión infinita es obtener generadores hermitianos j i , k i para el álgebra de Lorentz Lie, de modo que la transformación de Lorentz Λ = mi i ( α i j i + β i k i ) es unitario ( Λ = mi i ( α i j i + β i k i ) ) .

En el libro de Zee sobre el grupo (página 434) tenemos la siguiente definición para k 1 ,

i k 1 = t X + X t ,

y luego afirma (página 436) que i k 1 es hermitiano.

Para mí, esto es completamente incorrecto, porque si i k 1 eran hermitianos, entonces k 1 sería anti-hermitiano, y entonces habríamos perdido el objetivo de usar una representación de dimensión infinita para obtener generadores hermitianos.

Además, el cálculo explícito muestra que k 1 es hermitiano y luego i k 1 es anti-hermitiano:

  • El adjunto de un operador A se define por (Hassani, Mathematical Physics, página 61): ψ | A | ψ = ψ | A | ψ .

Entonces A es hermítica (es decir, autoadjunta) si

ψ | A | ψ = ψ | A | ψ .

  • En el espacio de Hilbert de funciones integrables al cuadrado, el producto interno está definido por (abajo X es un cuatro vector)
    F | gramo = F ( X ) gramo ( X ) d X .

Integrando por partes y suponiendo como de costumbre que ψ ( X ) , ψ ( X ) 0 cuando X b o tu norte d a r y , tu consigues eso

ψ | k 1 | ψ = ψ | k 1 | ψ .

Soy muy reacio a creer que Zee lo está haciendo mal, así que pregunto si mi razonamiento es correcto o me estoy perdiendo algún paso crucial.

Hay un problema convencional en el que a algunas personas les gusta que sus generadores sean anti-hermitianos (es decir, anti-hermitianos). T genera el unitario mi T ) y a algunas personas les gusta que sus generadores sean hermitianos (es decir, hermitianos T genera el unitario mi i T ). ¿Estás seguro de que esto no es solo una diferencia en la convención? Comparación de factores de i y 1 a través de diferentes fuentes es a menudo difícil debido a esto.
No parece un tema meramente convencional, pues en las mismas páginas afirma que j i es hermitiano, entonces j i es hermitiano y k i anti-ermitano. Si fuera solo un problema convencional, esperaría que los generadores fueran todos hermíticos o antihermitanos.
En ese caso, ¿está seguro de que la representación que Zee realmente está describiendo es la representación unitaria en el espacio de estados, y no la representación definitivamente no unitaria en los campos? En este último, los impulsos son antiunitarios, consulte, por ejemplo, physics.stackexchange.com/q/669780/50583
En esa página está usando el operador diferencial completo definido por el generador. Tengo entendido que los generadores definidos a través de operadores diferenciales se usan solo para el espacio de estados. Si fuera la representación en los campos, ¿no debería usar la representación matricial habitual (que no es unitaria)?
También en mi publicación hice el cálculo explícito que muestra que k i no es anti-ermitano
No entiendo lo que t está en este contexto. El L 2 el espacio considera la variable X solo.
X es el cuatro vector entonces X = ( X 0 , X 1 , X 2 , X 3 ) dónde X 0 = t y X 1 = X , X 2 = y , X 3 = z
y luego
i k 1 = X 0 X 1 + X 1 X 0
No tiene sentido. L 2 se refiere sólo a variables espaciales. en ese contexto t es un parámetro externo t no es un operador actuando en el espacio de Hilbert. El campo vectorial k 1 que escribiste no es el generador de la simetría de Lorentz en el espacio de Hilbert.
Se puede definir un espacio de Hilbert incluyendo el t coordenada, sin embargo ese espacio de Hilbert no es el de la teoría cuántica. Allí, el espacio de Hilbert solo se refiere a variables espaciales/momentos espaciales si la partícula es masiva, o los momentos espaciales si no tiene masa.
¿Ha revisado WP o el Teorema 10.5 de Wu-Ki Tung? Los campos cuánticos transforman la no unitaridad, pero los modos del oscilador unitariamente. Las "funciones de onda" proporcionan la conexión/conversión torcida.
Su "entendimiento es que los generadores definidos a través de operadores diferenciales se usan solo para el espacio de estados" es incorrecto. Estos operadores diferenciales actúan sobre un cuadrivector y sus composiciones. Claramente no en el espacio de los estados (Fock).
@CosmasZachos, gracias, tienes razón, el espacio de Hilbert de funciones cuadradas integrables no puede ser el espacio de estados. Sin embargo, parece que esos operadores diferenciales proporcionan una representación de dimensión infinita. Esto está claramente establecido en el libro Maggiore, últimas 10 líneas en la página 30. Yo diría que la afirmación de Zee es que k 1 es anti-hermitian sigue siendo incorrecto: debe mencionar lo que quiere decir con "hermitian", porque el objeto aquí es un operador diferencial que actúa sobre funciones (no una matriz), por lo que debe mencionar qué es el producto escalar.
Por lo que sé, los operadores diferenciales suelen actuar en el espacio de Hilbert de funciones cuadradas integrables y el producto escalar suele definirse como en mi publicación. Si eso es correcto entonces k 1 no es anti-ermitano. Acabo de enterarme de que en la página 52 del libro de Maggiore, el producto escalar se define de manera diferente (la integral está solo en la variable espacial y aparece la derivada del tiempo). Con esa definición de producto escalar, afirma que los operadores diferenciales anteriores son hermitianos.

Respuestas (1)

Creo que las preguntas vinculadas y el artículo y los libros de WP resuelven la pregunta del título, que la mayoría de los que respondieron identificaron como el corazón de la misma, al señalar los campos cuánticos que se transforman en la irrep no unitaria de dimensión finita del grupo de Lorentz, mientras que los estados/partículas en el irrep de dimensión infinita del mismo grupo, y cómo los dos se engranan/reempaquetan y hacen la transición entre sí.

Sin embargo, creo que el núcleo de su pregunta es el malentendido de la realización diferencial que está utilizando para los impulsos antihermitanos. k i , p.ej

i k 1 = t X + X t   .
Esta es una abreviatura de campo vectorial que resume la acción del impulso en los irreps 4D no unitarios del impulso de Lorentz, es decir, 4 vectores ( t , X , y , z ) .

Para simplificar, dado que solo está actuando en el subespacio 2D (t,x) , trunquemos (y,z) ,

( t X ) = mi ζ σ 1 ( t X ) = ( aporrear ζ pecado ζ pecado ζ aporrear ζ ) ( t X ) ,
dónde ζ es la rapidez y c=1 . Observa que esta transformación definitivamente no es unitaria, lo cual está bien: solo tiene la intención de preservar el intervalo t 2 X 2 , y no un ingenuo producto escalar euclidiano positivo-definido.

La transformación infinitesimal es

( t , X ) T = ( t , X ) T ζ ( X , t ) T + O ( ζ 2 ) .
Entonces, por ejemplo, un campo escalar transformado es
ϕ ( t , X , y , z ) = ϕ ( t , X , y , z ) ζ ( X t + t X ) ϕ ( t , X , y , z ) + O ( ζ 2 ) .
Pero, "en el fondo", aprecia que solo está transformando 4 vectores, no los vectores de dimensión infinita paradigmáticos del espacio QM Hilbert elemental (donde i X realmente se traduce a las familiares matrices de Heisenberg de dimensión infinita ).

Para campos no escalares, como el campo del espinor, por ejemplo, se revuelven aún más los cuatro componentes del 4-spinor, nuevamente una representación finita, no infinita, del grupo de Lorentz, a través del incremento igualmente hermitiano, no antihermitiano, que involucra el Aumento de la matriz γ . Recuerdo el texto de Schwartz, al final de la página 171, ilustra claramente cómo esto encaja perfectamente con la invariancia de Lorentz de ψ ¯ ψ !

Entonces, la conclusión es que su realización diferencial actúa de manera contraintuitiva en representaciones de dimensión finita y difiere dramáticamente de la L 2 formalismo en el que se está enfocando (así como la matriz de dimensión infinita independientemente de los operadores de creación y aniquilación que describen los estados de las partículas). No deseo involucrarme en la libidinosidad pedagógica o en la inoptimidad de los textos que está discutiendo. (Tengo mis opiniones sombrías personales sobre uno de ellos, hace medio siglo, incluso antes de que fuera escrito...).

¿Puedo preguntarle cuál es, en su opinión, un buen libro introductorio sobre QFT con un enfoque en los conceptos de grupo?
Sin embargo, debo decir que también Schwartz afirma que L m v = i ( X m X v X v X m ) es una representación de dimensión infinita del grupo de Lorentz (página 161)
Me temo que tiene razón; Matt también promulga tonterías en ocasiones. Aún así, entiende bien el punto sobre los espinores, al final de la página 171, donde la no unitaridad es esencial. Weinberg tiene mucho, pero es una lectura excepcionalmente formal y hostil. Elige desde aquí . El libro de teoría de grupos de Wu-Ki Tung, mencionado anteriormente, es el más amigable.
Una forma de compromiso de leer la declaración de Matt con aceptación es notar que esta acción en el espacio de funciones es reducible. Es decir, como se muestra arriba, la acción del grupo de Lorentz transforma las variables de la función, manteniéndolo en la misma forma de función. La acción pasa a ser la misma para cualquier otra función que, en cierto sentido, ϕ ( t , X , y , z ) no ve directamente. La estructura es un producto directo infinito de irreps 4D.
Mi sentido es no obsesionarme con declaraciones formales simplistas/elisivas ocultas, sino inspeccionar lo que realmente sucede con objetos básicos simples, que es lo que el lenguaje abstracto hiperformal se esfuerza por encapsular.
@Andrea El espacio de funciones con valores de 4 vectores es de dimensión infinita. La representación del grupo de Lorentz (o Poincaré) en este espacio no es unitaria, y es "en espíritu" solo la representación de 4 vectores de dimensión finita como explica Cosmas aquí, pero por supuesto es técnicamente de dimensión infinita porque el espacio de funciones es. Tenga cuidado: solo porque cada representación unitaria debe ser de dimensión infinita, esto no significa que cada representación de dimensión infinita deba ser unitaria.
¿Cuál es el producto escalar que debo usar para mostrar que esos generadores no son hermitianos (y luego la representación del grupo no es unitaria)?
No estoy seguro, nunca había visto tal cosa; tal vez Gilmore se moleste en discutirlo. Podría construirse alrededor del intervalo. Pero recuerde, todas las declaraciones habituales son sobre irreps , por lo tanto, las matrices no son sumas directas disjuntas de generadores de irreps, por lo que primero debe introducir un esquema para mapear su realización a una representación matricial en el espacio de funciones.
Gracias a todos los que respondieron que me ayudaron a tener (con suerte) una mejor comprensión. El mensaje que tomo es que esa representación es en realidad una representación del álgebra de Lorentz, técnicamente es de dimensión infinita, pero no es irreducible. Los irrep son los proporcionados como ejemplo en Wi-Ki Tung